Converting partial derivative w.r.t. T to partial derivative w.r.t. 1/T

Click For Summary
The discussion centers on proving the operator identity T * (∂/∂T) = (-1/T) * (∂/∂(1/T)). The derivation begins by defining F = 1/T and showing that the derivatives with respect to T and F yield equivalent results. A key point is the application of the chain rule to generalize the identity for any suitable function g(T). This clarification resolves concerns about dropping F in the operator context, confirming that the identity holds universally. The conclusion emphasizes the simplicity of the relationship once understood correctly.
knulp
Messages
6
Reaction score
0
Hi, I have a question about a certain step in the following problem/derivation, which you'll see in square brackets:

Show that T * (\partial/ \partialT) = (-1/T) * (\partial/ \partial(1/T))
["\partial/\partialT" is the operator that takes the partial derivative of something with respect to T]

Showing that this is true is a little tricky. For example, we can define F = 1/T. Then (\partialF/ \partialT) = -1/T^2 and (\partialF/ \partialF) = 1. So we can write

(\partialF/ \partialT) = (-1/T^2) (\partialF/ \partialF).

[In the next step he drops the F, so it's now an operator for an arbitrary function, but still with respect to F… Is this really okay?]

(\partial/ \partialT) = (-1/T^2)(\partial/ \partialF)

= (-1/T^2) (\partial/ \partial(1/T)).

Multiplying by T, T(\partial/ \partialT) = (-1/T)(\partial/ \partial(1/T)) and we're done.
 
Physics news on Phys.org
[In the next step he drops the F, so it's now an operator for an arbitrary function, but still with respect to F… Is this really okay?]


The operator identity holds by definition if, when acting on suitable functions, both sides of the identity give the same result. So in my understanding it is not quite right to conclude the operator identiy from only having shown that when acting on F they give the same result. However you can easily cure this. It is just the chain rule. Let g(T) be some function, F(T) = 1/T,. then

<br /> \frac{\partial g}{\partial T} = \frac{\partial F}{\partial T}\frac{\partial g}{\partial F} = -\frac{1}{T^2}\frac{\partial g}{\partial F}<br />

This is true for any suitable function g so

<br /> \frac{\partial }{\partial T} = -\frac{1}{T^2}\frac{\partial }{\partial F}<br />

or

<br /> T\frac{\partial }{\partial T} = -\frac{1}{T}\frac{\partial }{\partial (1/T)}<br />
 
Last edited:
Thanks, it makes complete sense now. So simple, doh!
 
Question: A clock's minute hand has length 4 and its hour hand has length 3. What is the distance between the tips at the moment when it is increasing most rapidly?(Putnam Exam Question) Answer: Making assumption that both the hands moves at constant angular velocities, the answer is ## \sqrt{7} .## But don't you think this assumption is somewhat doubtful and wrong?

Similar threads

  • · Replies 1 ·
Replies
1
Views
1K
Replies
9
Views
2K
  • · Replies 28 ·
Replies
28
Views
3K
Replies
10
Views
3K
  • · Replies 4 ·
Replies
4
Views
1K
  • · Replies 5 ·
Replies
5
Views
1K
  • · Replies 4 ·
Replies
4
Views
2K
  • · Replies 53 ·
2
Replies
53
Views
3K
  • · Replies 8 ·
Replies
8
Views
2K
  • · Replies 1 ·
Replies
1
Views
1K